- PowerScore Staff
- Posts: 5972
- Joined: Mar 25, 2011
- Fri Jan 20, 2017 12:00 am
#45461
Complete Question Explanation
(The complete setup for this game can be found here: lsat/viewtopic.php?t=4080)
The correct answer choice is (A)
Answer choice (A) is the correct answer choice.
Answer choice (B) is incorrect because cage W has three animals, which is impossible according to the first rule.
Answer choice (C) is incorrect because Y has five animals, which is impossible according to the first rule.
Answer choice (D) is incorrect because in the scenario presented there are four remaining snakes to be placed, but no more lizards to put with the snakes.
Answer choice (E) is incorrect because cages with gerbils must have hamsters.
(The complete setup for this game can be found here: lsat/viewtopic.php?t=4080)
The correct answer choice is (A)
Answer choice (A) is the correct answer choice.
Answer choice (B) is incorrect because cage W has three animals, which is impossible according to the first rule.
Answer choice (C) is incorrect because Y has five animals, which is impossible according to the first rule.
Answer choice (D) is incorrect because in the scenario presented there are four remaining snakes to be placed, but no more lizards to put with the snakes.
Answer choice (E) is incorrect because cages with gerbils must have hamsters.
Dave Killoran
PowerScore Test Preparation
Follow me on X/Twitter at http://twitter.com/DaveKilloran
My LSAT Articles: http://blog.powerscore.com/lsat/author/dave-killoran
PowerScore Podcast: http://www.powerscore.com/lsat/podcast/
PowerScore Test Preparation
Follow me on X/Twitter at http://twitter.com/DaveKilloran
My LSAT Articles: http://blog.powerscore.com/lsat/author/dave-killoran
PowerScore Podcast: http://www.powerscore.com/lsat/podcast/